PT85.S2.Q9 - Pharmaceutical manufacturers

Star PlatinumStar Platinum Member
edited November 2022 in Logical Reasoning 83 karma

I understand why A is correct, but why is E wrong? Is it because E does the opposite of weaken and just strengthens the argument?

Comments

  • 5Fennel LSAT5Fennel LSAT Member
    192 karma

    Good morning,

    E doesn't have any effect on the argument because the premises concern the number of visits per representative, and not the number of visits per physician, and thereby does not address the flawed necessary assumption in the argument. From the premises, it cannot be determined if invididual physicians received more or fewer visits.

    The argument depends on the assumption that fewer visits per representative indicates that the promotion is counterproductive, and there are many ways that the assumption can be negated, which A directly addresses - fewer visits yes, but more time to each physician can be interpreted as not counterproductive.

  • Star PlatinumStar Platinum Member
    83 karma

    That was so helpful. Thank you so much!

Sign In or Register to comment.